• No results found

Farey Fractions

N/A
N/A
Protected

Academic year: 2022

Share "Farey Fractions"

Copied!
40
0
0

Loading.... (view fulltext now)

Full text

(1)

U.U.D.M. Project Report 2017:24

Examensarbete i matematik, 15 hp Handledare: Andreas Strömbergsson Examinator: Jörgen Östensson

Juni 2017

Department of Mathematics

Farey Fractions

Rickard Fernström

(2)
(3)

Farey Fractions Uppsala University

Rickard Fernstr¨ om

June 22, 2017

(4)

1 Introduction

The Farey sequence of order n is the sequence of all reduced fractions be- tween 0 and 1 with denominator less than or equal to n, arranged in order of increasing size. The properties of this sequence have been thoroughly in- vestigated over the years, out of intrinsic interest. The Farey sequences also play an important role in various more advanced parts of number theory.

In the present treatise we give a detailed development of the theory of Farey fractions, following the presentation in Chapter 6.1-2 of the book

MNZ = I. Niven, H. S. Zuckerman, H. L. Montgomery, ”An Introduction to the Theory of Numbers”, fifth edition, John Wiley & Sons, Inc., 1991, but filling in many more details of the proofs.

Note that the definition of ”Farey sequence” and ”Farey fraction” which we give below is apriori different from the one given above; however in Corollary 7 we will see that the two definitions are in fact equivalent.

2 Farey Fractions and Farey Sequences

We will assume that a fraction is the quotient of two integers, where the denominator is positive (every rational number can be written in this way).

A reduced fraction is a fraction where the greatest common divisor of the numerator and denominator is 1. E.g. 3.5

4 is not a fraction, but 7

8 is both a fraction and a reduced fraction (even though we would normally say that

3.5 4 = 7

8). Also −1

−2 and 0

−1 are not fractions, since their denominators are negative.

We will construct a table in the following way, where the fractions in each row of the table are in the specified order. The 1st row only contains the fractions 0

1 and 1

1. If the nth row has been constructed, then the (n + 1)st row is constructed by copying the nth row and then for each pair of consec- utive fractions a

b and a0

b0 that exist in the nth row that satisfy b + b0 ≤ n + 1,

(5)

the fraction a + a0

b + b0 is inserted in the (n + 1)st row between a

b and a0

b0. For ex- ample, the 2nd row is constructed by copying the 1st row and then inserting

0 + 1 1 + 1 = 1

2 between 0

1 and 1

1, so that the 2nd row is 0 1, 1

2, 1

1. When con- structing the 3rd row, 0 + 1

1 + 2 = 1

3 is inserted between 0

1 and 1

2 and 1 + 1 2 + 1 = 2

3 is inserted between 1

2 and 1

1, so that the 3rd row becomes 0 1, 1

3, 1 2, 2

3, 1 1. When constructing the 4th row, 0 + 1

1 + 3 = 1

4 is inserted between 0

1 and 1 3 and 2 + 1

3 + 1 = 3

4 is inserted between 2

3 and 1

1, but 1 + 1 3 + 2 = 2

5 is not inserted between 1

3 and 1

2 because 5 6≤ 4 and 1 + 2 2 + 3 = 3

5 is not inserted between 1 2 and 2

3 because 5 6≤ 4. So the 4th row is 0 1, 1

4, 1 3, 1

2, 2 3, 3

4, 1

1. The first six rows in the table are:

0 1

1 1 0

1

1 2

1 1 0

1

1 3

1 2

2 3

1 1 0

1

1 4

1 3

1 2

2 3

3 4

1 1 0

1

1 5

1 4

1 3

2 5

1 2

3 5

2 3

3 4

4 5

1 1 0

1 1 6

1 5

1 4

1 3

2 5

1 2

3 5

2 3

3 4

4 5

5 6

1 1

.

Definition 1 (Farey sequence). The sequence of fractions in the nth row in the above table is called the Farey sequence of order n.

Definition 2 (Farey fraction of order n). A Farey fraction of order n is a fraction in the Farey sequence of order n.

(6)

Definition 3 (Farey fraction). A Farey fraction is a Farey fraction of some order, i.e. a fraction in the Farey table.

Theorem 1 (Theorem 6.1 and Corollary 6.3 in MNZ). If a

b and a0 b0 are consecutive fractions in the nth row with a

b to the left of a0

b0, then a0b−ab0 = 1.

The fractions in the nth row are also listed in order of their size (in strictly ascending order).

Proof. Base case (n = 1): 1 · 1 − 0 · 1 = 1 and the fractions in the first row are clearly listed in order of their size (in strictly ascending order).

Induction hypothesis: Assume for some n ∈ N>0that if a

b and a0

b0 are fractions in the nth row with a

b to the left of a0

b0, then a0b − ab0 = 1. Also assume the fractions in the nth row are listed in order of their size (in strictly ascending order).

Induction step: We want to show that the fractions in the (n + 1)st row are listed in order of their size (in strictly ascending order). We know that the (n + 1)st row is constructed by copying the nth row and then for each pair of consecutive fractions in the nth row, insert 1 or 0 fractions between those fractions and we also know by the induction hypothesis that the fractions in the nth row are listed in order of their size (in strictly ascending order). So it’s sufficient to show that for each pair of consecutive fractions p

q, r

s in the nth row with p

q to the left, p

q < p + r q + s < r

s. So let p

q and r

s be consecutive fractions in the nth row with p

q to the left. Then p

q < p + r

q + s ⇐⇒ p(q + s)

q(q + s) < q(p + r) q(q + s)

⇐⇒ p(q + s) < q(p + r)

⇐⇒ rq − ps > 0.

But the last inequality holds because it follows from the induction hypoth- esis that rq − ps = 1. Hence p

q < p + r

q + s. Similarly it can be shown that

(7)

p + r q + s < r

s. So the fractions in the (n + 1)st row are listed in order of their size (in strictly ascending order) and from that it follows that no fraction can appear twice in the (n + 1)st row.

Let a

b and a0

b0 be 2 consecutive fractions in the (n + 1)st row with a b to the left of a0

b0. We want to show that a0b − ab0 = 1. If they are also con- secutive fractions in the nth row with a

b to the left, then it follows from the induction hypothesis that a0b − ab0 = 1. So assume they are not consecutive fractions in the nth row with a

b to the left.

We want to show it can’t be the case that both a

b and a0

b0 exist in the nth row. Assume for contradiction that both a

b and a0

b0 exist in the nth row. If they are not consecutive fractions in the nth row, then there is some frac- tion in the nth row somewhere between them, say p

q. But because of how rows are constructed, p

q must be between a

b and a0

b0 in the (n + 1)st row, which contradicts a

b and a0

b0 being consecutive fractions in the (n + 1)st row.

Assume instead for contradiction that a

b and a0

b0 are consecutive fractions in the nth row, but with a0

b0 to the left. Either no fraction was added between them when constructing the (n + 1)st row, in which case a

b and a0

b0 will be consecutive fractions in the (n + 1)st row, but with a0

b0 to the left. This leads to a contradiction. If instead a fraction was added between a

b and a0 b0 when constructing the (n + 1)st row, then a

b and a0

b0 are not even consecutive frac- tions in the (n + 1)st row, which is a contradiction. So it can’t be the case that both a

b and a0

b0 exist in the nth row when they are consecutive fractions in the (n + 1)st row with a

b to the left and they are not consecutive fractions

(8)

in the nth row with a

b to the left.

We want to show that it can’t be the case that neither of the fractions a b and a0

b0 exist in the nth row. Assume or contradiction that neither of the fractions a

b and a0

b0 exist in the nth row. Since they exist in the (n + 1)st row, it must be the case that they were both inserted between fractions in the nth row.

But because at most 1 fraction is inserted between each pair of consecutive fractions, they must have been inserted between distinct pairs of fractions in the nth row. But then it’s clear that some fraction must exist between a

b and a0

b0 in the (n + 1)st row, which contradicts them being consecutive fractions in the (n + 1)st row. So it’s impossible that neither of the fractions a

b and a0

b0 exist in the nth row when they are consecutive fractions in the (n + 1)st row with a

b to the left.

The remaining case is when one of the fractions exist in the nth row but the other doesn’t. Assume it’s the left fraction a

b that exists in the nth row (if instead it’s the right fraction that exists in the nth row, then the proof can be done analogously). The right fraction a0

b0 must have been inserted in the (n + 1)st row between a

b and the fraction in the nth row directly to the right of a

b, say p

q. Then the nth and (n + 1)st row will look something like this:

Row n: ... a b

p q ...

Row n + 1: ... a b

a + p b + q = a0

b0 p q ...

.

Then

a0b − ab0 = (a + p)b − a(b + q) = pb − aq

= 1. {by induction hypothesis}

(9)

Corollary 2 (Corollary 6.2 in MNZ). Every fraction a

b in the table is in reduced form, i.e. gcd(a, b) = 1.

Proof. Let n ∈ N>0. We want to show every fraction a

b in the nth row has gcd(a, b) = 1.

Let a

b be a fraction in the nth row. Since there is more than 1 fraction in the nth row, there is a fraction, say p

q next to a

b in the nth row. Assume p

q is to the right of a

b (if it is to the left of a

b, then the proof can be done analogously). By Theorem 1, pb−aq = 1. Consider the diophantine equation bx + ay = 1. It has a solution iff gcd(a, b) = 1. But x = p, y = −q is a solution, so gcd(a, b) = 1.

Theorem 3. ∀n ∈ N>0: if a

b and a0

b0 are consecutive fractions in the nth row, then b + b0 ≥ n + 1.

Proof. For n = 1, 1 + 1 = 2 ≥ 1 + 1, so it’s true for n = 1.

Induction hypothesis: Assume it’s true for some n ∈ N>0. Induction step: We want to show it’s true for n + 1.

Let a

b and a0

b0 be 2 consecutive fractions in the (n+1)st row with a

b to the left.

If they are also consecutive fractions in the nth row then by the induction hypothesis, b + b0 ≥ n + 1. It can’t be the case that b + b0 = n + 1, because if that were the case then b + b0 ≤ n + 1, so in the (n + 1)st row a fraction should have been inserted between a

b and a0

b0, but then a

b and a0

b0 can’t be consecutive fractions in the (n + 1)st row, a contradiction. So b + b0 > n + 1, i.e. b + b0 ≥ n + 2.

Otherwise if they are not consecutive fractions in the nth row, exactly 1 of the fractions appear in the nth row like in Theorem 1. Say a

b appears in the nth row and p

q is the fraction in the nth row directly to the right of a

b. Then a0 b0 is the fraction ”between” a

b and p

q, so b0 = b + q. By the induction hypothesis, b+q ≥ n+1. And hence b+b0 = b+b+q ≥ 1+b+q ≥ 1+(n+1) = n+2.

(10)

Theorem 4. Let n ∈ N>0 and let p

q be a fraction in the nth row. Then q ≤ n with equality iff p

q does not exist in any previous row.

Proof. We will begin by showing the first part, i.e. ∀n ∈ N>0: if p q is a fraction in the nth row, then q ≤ n.

For n = 1 the only fractions are 0

1 and 1

1, and 1 ≤ 1.

Induction hypothesis: Assume for some n ∈ N>0 that for every fraction p q in the nth row, q ≤ n.

Induction step: Let p

q be a fraction in the (n + 1)st row. We want to show that q ≤ n + 1.

Case 1 : p

q exists in the nth row as well. Then by induction hypothesis, q ≤ n < n + 1.

Case 2 : p

q does not exist in the nth row. Then p

q must lie between 2 fractions that exist in the nth row, say a

b and a0

b0. Then q = b + b0 and because of the rules of how rows are constructed, b + b0 ≤ n + 1, so q ≤ n + 1.

Now we want to prove the second part, i.e. ∀n ∈ N>0: if p

q is a fraction in the nth row, then q = n iff p

q does not exist in any previous row.

For n = 1, we see that q = 1 and p

q does not exist in any previous row, so the theorem holds for n = 1.

Say n > 1 and let p

q be a fraction in the nth row. If p

q exists in a previous row, then since rows are constructed by copying the previous row, p

q must exist in the (n − 1)st row. This means we are in case 1 of the proof of the previous part of this theorem, so q < n, i.e. q 6= n.

If instead p

q doesn’t exist in a previous row, then we are in case 2 of the proof of the previous part of this theorem, i.e. p

q lies between the fractions a b and a0

b0 that exist in the (n − 1)st row. Then q = b + b0 ≤ n. But because of

(11)

Theorem 3, b + b0 ≥ n. Since q ≤ n and q ≥ n we must have that q = n.

Lemma 5. Let n ∈ N>0 and let x ∈ N be such that 0 ≤ x ≤ n + 1 and gcd(x, n + 1) = 1. Let a

b be the greatest fraction in the nth row that is less than x

n + 1 and let a0

b0 be the smallest fraction in the nth row that is greater than x

n + 1. Then a + a0 = x and b + b0 = n + 1.

Proof. It can’t be the case that x

n + 1 is fraction in the nth row, since that would contradict Theorem 4 (n + 1 6≤ n). So then a

b and a0

b0 are consecutive fractions in the nth row with a

b to the left and a

b < x

n + 1 < a0 b0. We see that a

b < x

n + 1 ⇐⇒ bx − a(n + 1) > 0 ⇐⇒ bx − a(n + 1) ≥ 1.

Consider the diophantine equation

bu − av = k,

where k := bx − a(n + 1) and u and v are the unknowns. It has a particular solution hu, vi = hka0, kb0i since

bka0− akb0 = k(a0b − ab0)

= k. by Theorem 1

We have that gcd(a, b) = 1 because of Corollary 2, so it has the general solution hu, vi = hka0 + ae, kb0 + bei, e ∈ Z. But we also know it has a particular solution hu, vi = hx, n + 1i, so ∃m ∈ Z : x = ka0+ am ∧ n + 1 = kb0 + bm. Let m ∈ Z be such that

kb0+ bm = n + 1 (1)

and

ka0+ am = x. (2)

(12)

We want to show that k = m = 1.

Since 0 ≤ x ≤ n + 1 and gcd(x, n + 1) = 1, it must be the case that x > 0.

We have that x

n + 1 < a0

b0 ⇐⇒ n + 1 x > b0

a0

⇐⇒ (n + 1)a0− b0x > 0

⇐⇒ (n + 1)a0− b0x ≥ 1

⇐⇒ (kb0+ bm)a0− b0(ka0 + am) ≥ 1 {by (1) and (2)}

⇐⇒ (a0b − ab0)m ≥ 1

⇐⇒ m ≥ 1. by Theorem 1

This shows that m ≥ 1. We also know that k ≥ 1. If we assume for contradiction that k > 1 or m > 1, then

b + b0 < bm + kb0

= n + 1, {by (1)}

but that’s impossible, since by Theorem 3, b + b0 ≥ n + 1. This shows that k ≤ 1 and m ≤ 1. Since also m ≥ 1 and k ≥ 1, it follows that k = m = 1. Equation (1) now becomes b + b0 = n + 1 and equation (2) becomes a + a0 = x.

Theorem 6 (Theorem 6.5 in MNZ). If 0 ≤ x ≤ y, gcd(x, y) = 1, then the fraction x

y appears in the yth and all later rows.

Proof. It is clear that if x

y appears in the yth row then it also appears in all later rows, so it is enough to show that x

y appears in the yth row.

If y = 1 then either x = 0 or x = 1. Both 0

1 and 1

1 appear in the 1st row, so the theorem holds for y = 1.

We want to show the theorem holds when y ≥ 2, i.e. when y = n + 1 for some n ∈ N>0.

Let n ∈ N>0, 0 ≤ x ≤ n + 1 and gcd(x, n + 1) = 1. If x = 0 or x = n + 1 then gcd(x, n + 1) = n + 1 > 1, so it must be the case that 0 < x < n + 1.

(13)

We want to show that x

n + 1 appears in the (n + 1)st row. Let a

b be the greatest fraction in the nth row that is less than x

n + 1 and let a0

b0 be the smallest fraction in the nth row that is greater than x

n + 1. By Lemma 5 we know that a + a0 = x and b + b0 = n + 1. From the first few lines of the proof of Lemma 5 we know that x

n + 1 does not exist in the nth row, so a b and a0

b0 must be consecutive fractions in the nth row. Because a + a0

b + b0 = x n + 1 and how rows are constructed, we know that x

n + 1 must be inserted in the (n + 1)st row between a

b and a0 b0.

Corollary 7 (Corollary 6.6 in MNZ). The nth row consists exactly of all reduced fractions a

b such that 0 ≤ a

b ≤ 1 and 0 < b ≤ n. The fractions are listed in order of their size.

Proof. Let n ∈ N>0 and let a

b be a reduced fraction such that 0 ≤ a b ≤ 1 and 0 < b ≤ n. We want to show that a

b appears in the nth row. Clearly 0 ≤ a ≤ b ≤ n. By Theorem 6 we have that a

b appears in the bth and all later rows, so a

b appears in the nth row. So every reduced fraction a b such that 0 ≤ a

b ≤ 1 and 0 < b ≤ n exist in the nth row, and they are listed in order of their size by Theorem 1. We want to show that the nth row doesn’t contain any other fractions than these. Let p

q be a fraction in the nth row. The fraction p

q must be reduced by Corollary 2. By Theorem 4 we have that q ≤ n and q > 0 by the definition of fractions. So 0 < q ≤ n.

Because rows are constructed by copying the previous row and then inserting fractions between the fractions and the fact that rows are ordered by size, we can never get a row with a fraction that is strictly greater than 1

1 or strictly less than 0

1. So 0 ≤ p q ≤ 1.

(14)

Theorem 8. If a

b and a0

b0 are Farey fractions with a0b − ab0 = 1 (and thus a

b < a0

b0), then a

b and a0

b0 are consecutive Farey fractions of some order.

Proof. Let a

b and a0

b0 be Farey fractions with

a0b − ab0 = 1 (3)

(and thus a b < a0

b0). Let c

d be a Farey fraction such that a b < c

d < a0 b0. We want to show that d > max(b, b0). From a

b < c

d and c d < a0

b0 it follows that (cb − da > 0

da0− cb0 > 0.

Let k := cb − da and l := da0− cb0. Then k and l are positive integers and (cb − da = k

da0− cb0 = l. (4)

Consider the diophantine equation

bx − ay = k. (5)

We know that

b(ka0) − a(kb0) = k(a0b − ab0)

= k. {by (3)}

Hence hx, yi = hka0, kb0i is a particular solution to (5). Because gcd(a, b) = 1 (this follows from Corollary 2), the general solution to (5) is hx, yi = hka0+ am, kb0+ bmi = kha0, b0i + mha, bi, m ∈ Z.

Consider the diophantine equation

−b0x + a0y = l. (6)

Since

−b0(al) + a0(bl) = l(a0b − ab0)

= l {by (3)}

(15)

it follows that hx, yi = hal, bli is a particular solution to (6). We have that gcd(a0, b0) = 1 because of Corollary 2, so the general solution to (6) is hx, yi = hal + a0n, bl + b0ni = lha, bi + nha0, b0i, n ∈ Z. We know that

a b

 , a0

b0



is a basis for the vector space R2, since

deta a0 b b0



= ab0− a0b

= −(a0b − ab0)

= −1 {by (3)}

6= 0.

But then every vector u v



∈ R2 can be written in a unique way as u v



= c1a

b



+c2a0 b0



, where c1, c2 ∈ R. From (4) it follows that hx, yi = hc, di is a particular solution to both (5) and (6). By looking at the general solution of (5) we see that c

d



= ka0 b0



+ ma b



for some m ∈ Z. By looking at the general solution of (6) we see that c

d



= la b



+ na0 b0



for some n ∈ Z. By comparing these 2 ways of writing c

d



and using the fact that

a b

 , a0

b0



is a basis for R2 we conclude that l = m and k = n. Thus

c d



= ka0 b0



+ la b



. But then

d = kb0+ lb ≥ b0 + b > max(b, b0) =: m.

In the above equations we used the fact that k and l are positive integers.

By Corollary 7 it follows that c

d does not exist in the mth row, since d > m.

But c

d was an arbitrary Farey fraction lying strictly between a

b and a0 b0. So no Farey fraction lying strictly between a

b and a0

b0 exist in the mth row, but both a

b and a0

b0 exist in the mth row. So a

b and a0

b0 must be consecutive Farey fractions in the mth row.

(16)

Corollary 9. Let a

b and a0

b0 be Farey fractions such that a b < a0

b0. Then a0b − ab0 = 1 iff a

b and a0

b0 are consecutive Farey fractions of some order.

Proof. ” =⇒ ”: Assume a0b − ab0 = 1. Then it follows from Theorem 8 that a

b and a0

b0 are consecutive Farey fractions of some order.

” ⇐= ”: Assume a

b and a0

b0 are consecutive Farey fractions of some order.

Then it follows from Theorem 1 that a0b − ab0 = 1.

Theorem 10 (Theorem 6.4 in MNZ). If a

b and a0

b0 are consecutive fractions in the nth row (with a

b to the left), then among all fractions p

q such that a

b < p q < a0

b0, p

q = a + a0

b + b0 is the unique fraction with smallest denominator.

Proof. Let n ∈ N>0 and let a

b and a0

b0 be consecutive fractions in the nth row, with a

b to the left. There are 2 possibilities.

Case 1 : b + b0 ≤ n + 1. Then b + b0 ≥ n + 1 by Theorem 3, so b + b0 = n + 1.

We have that a + a0

b + b0 will be inserted in the (n + 1)st row between a

b and a0 b0. Case 2 : b + b0 > n + 1. Then for all integers k such that n ≤ k < b + b0:

a

b and a0

b0 will be consecutive fractions in the kth row and a + a0

b + b0 will be inserted between a

b and a0

b0 in the (b + b0)th row. This is because the kth row is constructed by copying the (k − 1)st row and inserting fractions between the fractions. If a

b and a0

b0 are consecutive fractions in the (k − 1)st row and a + a0

b + b0 was not inserted between them in the kth row (i.e. b + b0 6≤ k), then a

b and a0

b0 will be consecutive fractions in the kth row as well. If a

b and a0 b0 are consecutive fractions in the (k − 1)st row then a fraction is only inserted between them in row k if b + b0 ≤ k.

(17)

In both cases we get that a

b, a + a0

b + b0 and a0

b0 are 3 consecutive fractions in the (b + b0)th row and that a

b and a0

b0 are consecutive fractions in the (b + b0− 1)st row.

Let x

y be a reduced fraction such that a b < x

y < a0

b0. Since a

b and a0 b0 are fractions in the nth row, it follows from Corollary 7 that 0 ≤ a

b ≤ 1 and 0 ≤ a0

b0 ≤ 1. Since a b < x

y < a0

b0, we have that 0 ≤ x

y ≤ 1. If we assume for contradiction that y < b+b0, then by Theorem 6,x

y appears in the (b+b0−1)st row. But that’s impossible since we know from Theorem 1 that fractions are listed in order of their size and there is no fraction strictly between a

b and a0 b0 in the (b + b0− 1)st row. So y ≥ b + b0, i.e. a reduced fraction that lies strictly between a

b and a0

b0 can’t have a denominator smaller than b + b0. Suppose x y has minimal denominator, i.e. y = b + b0. By Theorem 6, x

y exists in the (b + b0)th row. But since a

b, a + a0

b + b0 and a0

b0 are 3 consecutive fractions in the (b + b0)th row and the (b + b0)th row is ordered by size by Theorem 1, the only fraction in the (b + b0)th row that lies strictly between a

b and a0

b0 is a + a0 b + b0. So x

y must be the fraction a + a0 b + b0. If you have a fraction p

q that lies strictly between a

b and a0

b0 that is not re- duced, then it can’t be the case that q ≤ b + b0, since then you could simplify

p

q into a reduced fraction where the denominator is strictly less than b + b0, but that’s impossible.

Say you have a fraction r

s that lies strictly between a

b and a0

b0. Then s ≥ b+b0, since if r

s is not reduced, then s > b + b0 and if r

s is reduced, then s ≥ b + b0. Also since a + a0

b + b0 exists in the (b + b0)th row it must be a reduced fraction because of Corollary 2. So the smallest denominator r

s can have is b + b0,

(18)

and in that case r

s must be a reduced fraction. So if you have a fraction that lies strictly between a

b and a0

b0 with smallest denominator, then it must be a reduced fraction with denominator b+b0(since b+b0is the smallest denomina- tor). But the only reduced fraction with denominator b + b0 that lies strictly between a

b and a0

b0 is a + a0

b + b0. So among all fractions that lie strictly between a

b and a0

b0, a + a0

b + b0 is the unique fraction with smallest denominator.

Proposition 11 (Problem 6.1.1 in MNZ). Let n be a positive integer such that n > 1 and let a

b and a0

b0 be the Farey fractions immediately to the left and the right of 1

2 respectively in the Farey sequence of order n. Then b = b0 = 1 + 2 · n − 1

2



, i.e. b is the greatest odd integer ≤ n. It is also true that a + a0 = b.

Proof. By Corollary 7 we know that 1

2 exists in the Farey sequence of order n since n ≥ 2. We will prove the Proposition by induction.

Base case (n = 2): Then we see from the Farey table on page 3 that a b is 0

1 and a0 b0 is 1

1. We have that 1 = 1, 1 is the greatest odd integer ≤ 2 and 0 + 1 = 1.

Induction hypothesis: Assume for some n ≥ 2 that if a

b and a0

b0 are the Farey fractions immediately to the left and the right of the fraction 1

2 respectively in the Farey sequence of order n, then b = b0 and b is the greatest odd integer

≤ n and a + a0 = b.

Induction step: Let a

b and a0

b0 be the Farey fractions immediately to the left and the right of the fraction 1

2 respectively in the Farey sequence of order n.

Let c

d and c0

d0 be the Farey fractions immediately to the left and the right of the fraction 1

2 respectively in the Farey sequence of order n + 1. We want to

(19)

show that d = d0and that d is the greatest odd integer ≤ n + 1 and c + c0 = d.

Case 1: b + 2 = b0+ 2 = n + 1 (b = b0 by induction hypothesis). Then when constructing the (n + 1)st row the fraction a + 1

b + 2 is inserted between a b and 1

2 and also the fraction a0+ 1

b0+ 2 is inserted between 1

2 and a0

b0. Then it must be the case that c

d is the fraction a + 1

b + 2 and c0

d0 is the fraction a0 + 1

b0+ 2, so then d = b + 2 and d0 = b0 + 2. Also from the induction hypothesis we see that b and b0 are odd, and since b + 2 = n + 1 we have that n = b + 1, i.e. n is even. From the induction hypothesis we know that b is the greatest odd integer ≤ n. Since d = b + 2 we need to show that b + 2 is the greatest odd integer ≤ n + 1, but this is obvious, since we know that n is even.

Since d = b + 2 and d0 = b0+ 2 and b = b0 by induction hypothesis, it follows that d = d0, so

d = d0

and d is the greatest odd integer ≤ n + 1. We also see that

c + c0 = a + 1 + a0+ 1 {c = a + 1 and c0 = a0+ 1}

= b + 2 {by induction hypothesis}

= d.

Case 2: b + 2 = b0 + 2 > n + 1 (b = b0 by induction hypothesis). Then no fraction will be inserted between a

b and 1

2 or between 1

2 and a0

b0 when constructing the (n + 1)st row. So c

d is the fraction a

b and c0

d0 is the fraction a0

b0. We know that

n + 1 < b + 2

≤ n + 2, {b ≤ n by Theorem 4}

so it follows that b + 2 = n + 2, i.e. b = n. But b is odd by the induction hypothesis, so n must be odd. We know from the induction hypothesis that b is the greatest odd integer ≤ n and d = b. We need to show that b is the greatest odd integer ≤ n + 1 But that is obvious.

(20)

Since b = b0 by induction hypothesis, b = d and b0 = d0 it follows that d = d0. So

d = d0 and d is the greatest odd integer ≤ n + 1.

We also see that

c + c0 = a + a0 {a = c and a0 = c0}

= b {by induction hypothesis}

= d.

There is no case where b + 2 = b0+ 2 < n + 1 because of Theorem 3.

Theorem 12 (Problem 6.1.9 in MNZ). For each Farey fraction a

b let C a b

 denote the circle in the plane of radius (2b2)−1 and center a

b, (2b2)−1 . These circles are called the Ford circles. The interior of a Ford circle contains no point of any other Ford circle and two Ford circles C a

b



and C  a0 b0



are tangent if and only if a

b and a0

b0 are consecutive Farey fractions of some order.

Proof. First assume that a

b and a0

b0 are distinct Farey fractions such that a

b < a0

b0. We want to show that neither of the Ford circlesC a b



andC  a0 b0



contain a point that is an interior point of the other Ford circle. C a b

 has radius

r := 1 2b2 and center

 a b, 1

2b2



=a b, r

.

(21)

C  a0 b0



has radius

s := 1 2(b0)2 and center

 a0 b0, 1

2(b0)2



= a0 b0, s

 .

Let d be the distance between the center ofC a b



and the center ofC  a0 b0

 . Then

d2 = a0 b0 −a

b

2

+ (s − r)2 = (a0b − ab0)2

b2(b0)2 + s2+ r2− 2rs. (7) In order to show that the Ford circlesC a

b



andC  a0 b0



don’t contain any interior points of the other Ford circle, it is sufficient to show that d ≥ r + s, i.e. d2 ≥ r2+ s2+ 2rs. Using (7) we see that we must show that

(a0b − ab0)2

b2(b0)2 + s2+ r2− 2rs ≥ r2+ s2+ 2rs

⇐⇒ (a0b − ab0)2

b2(b0)2 ≥ 4rs = 1 b2(b0)2

⇐⇒ (a0b − ab0)2 ≥ 1. (8)

It can’t be the case that a0b − ab0 = 0, since then a0b − ab0 = 0

⇐⇒ a0b = ab0

⇐⇒ a0 b0 = a

b, {divide both sides by bb0}

which contradicts the Farey fractions being distinct. But then a0b − ab0 is a non-zero integer, hence its square must be greater than or equal to 1, i.e. (8) is true. So the Ford circles C a

b



and C  a0 b0



don’t contain any interior

(22)

points of the other Ford circle and d ≥ r + s.

We want to show that C a b



and C  a0 b0



are tangent iff a

b and a0 b0 are consecutive Farey fractions of some order. If a

b and a0

b0 are the same Farey fraction, then the Ford circles C a

b



and C  a0 b0



are the same and are therefore not tangent and a

b and a0

b0 are not consecutive Farey fractions of some order, so the Theorem holds in that case. So assume a

b and a0

b0 are dis- tinct. Without loss of generality assume that a

b < a0

b0 like before. We know that C a

b



and C  a0 b0



are tangent if and only if d = r + s or d = |r − s|.

Since r and s are positve, it follows that |r − s| ≤ max(r, s) < r + s. But before we showed that d ≥ r + s, so it can’t be the case that d = |r − s|.

Therefore C a b



and C  a0 b0



are tangent if and only if d = r + s. But d = r + s iff d2 = r2+ s2+ 2rs. Using (7) we see that

d2 = r2+ s2+ 2rs

⇐⇒ (a0b − ab0)2

b2(b0)2 + s2+ r2− 2rs = r2+ s2+ 2rs

⇐⇒ (a0b − ab0)2

b2(b0)2 = 4rs = 1 b2(b0)2

⇐⇒ (a0b − ab0)2 = 1

⇐⇒ a0b − ab0 = ±1.

But it can easily be shown that a b < a0

b0 iff a0b−ab0 > 0, and since we assumed that a

b < a0

b0, it must be the case that a0b − ab0 > 0. Therefore a0b − ab0 = ±1 ⇐⇒ a0b − ab0 = 1.

From Corollary 9 it follows that a0b − ab0 = 1 ⇐⇒ a

b and a0

b0 are consecutive

(23)

Farey fractions of some order. SoC a b



andC  a0 b0



are tangent if and only if a

b and a0

b0 are consecutive Farey fractions of some order.

Remark 1. We will now use an alternative method to show that if a

b and a0 b0 are consecutive Farey fractions of some order, then C a

b



and C  a0 b0

 are tangent. When using this method we will find explicitly the coordinates for the intersection of the Ford circles.

Proof. Let a

b and a0

b0 be consecutive Farey fractions of order n, with a b < a0

b0. We want to show that C a

b



and C  a0 b0



are tangent. The equation for C a

b

 is

 x − a

b

2

+

 y − 1

2b2

2

= 1

4b4 (9)

and the equation for C  a0 b0

 is

 x −a0

b0

2

+



y − 1 2(b0)2

2

= 1

4(b0)4. (10) Then the x-coordinate of center ofC a

b



is less than (i.e. to the left of) the x-coordinate of the center of C  a0

b0



. Let L be the line that goes through

(24)

the center of C a b



and the center ofC  a0 b0



. It’s slope is

 1

2(b0)2 − 1 2b2

 / a0

b0 − a b



= b2− (b0)2 2b2(b0)2



/ a0b − ab0 bb0



= b2− (b0)2 2b2(b0)2

 / 1

bb0



{by Theorem 1}

= b2− (b0)2 2bb0 . The equation for L is y − 1

2(b0)2 = b2− (b0)2 2bb0

 x − a0

b0



. We want to see where L intersects C  a0

b0



, so we plug the equation for L into (10) and get

 x − a0

b0

2

+ b2 − (b0)2 2bb0

2 x − a0

b0

2

= 1

4(b0)4

⇐⇒ 1 + b2− (b0)2 2bb0

2!

 x −a0

b0

2

= 1

4(b0)4

⇐⇒  4b2(b0)2

4b2(b0)2 + (b2)2− 2b2(b0)2+ ((b0)2)2 4b2(b0)2

  x −a0

b0

2

= 1

4(b0)4

⇐⇒  b2+ (b0)2 2bb0

2 x −a0

b0

2

= 1

4(b0)4

⇐⇒

 x − a0

b0

2

= b2

(b0)2(b2+ (b0)2)2

⇐⇒ x = a0 b0 ±

s

b2

(b0)2(b2+ (b0)2)2

⇐⇒ x = a0

b0 ± b

b0(b2+ (b0)2).

We only care about the left intersection, i.e. where x = a0

b0 − b

b0(b2 + (b0)2).

(25)

If we plug this x-value into the equation for L and solve for y we get y = 1

2(b0)2 +b2− (b0)2

2bb0 · −b b0(b2+ (b0)2)

= 1

2(b0)2 − b2− (b0)2 2(b0)2(b2+ (b0)2)

= b2+ (b0)2

2(b0)2(b2+ (b0)2)− b2− (b0)2 2(b0)2(b2+ (b0)2)

= 1

b2+ (b0)2. So x = a0

b0 − b

b0(b2 + (b0)2) and y = 1

b2+ (b0)2 is the left intersection of L and C  a0

b0



. We want to show that this point also belongs to C a b



. Plugging

(26)

the x-value and y-value into the left hand side of (9) gives

 a0

b0 − b

b0(b2+ (b0)2) −a b

2

+

 1

b2+ (b0)2 − 1 2b2

2

= a0b − ab0

bb0 − b

b0(b2+ (b0)2)

2

+

 2b2

2b2(b2+ (b0)2) − b2+ (b0)2 2b2(b2+ (b0)2)

2

= 1

bb0 − b b0(b2+ (b0)2)

2

+

 b2− (b0)2 2b2(b2+ (b0)2)

2

{by Theorem 1}

=

 b2 + (b0)2

bb0(b2+ (b0)2) − b2 bb0(b2+ (b0)2)

2

+

 b2 − (b0)2 2b2(b2+ (b0)2)

2

=

 2b(b0)3 2b2(b0)2(b2+ (b0)2)

2

+

 b2(b0)2− (b0)4 2b2(b0)2(b2+ (b0)2)

2

= 4b2(b0)6+ b4(b0)4− 2b2(b0)6+ (b0)8 4b4(b0)4(b2+ (b0)2)2

= (b0)4(b2+ (b0)2)2 4b4(b0)4(b2 + (b0)2)2

= 1 4b4,

which means that a0

b0 − b

b0(b2+ (b0)2), 1 b2+ (b0)2



belongs toC a b



,C  a0 b0



and L, so it must be the case that C a b



and C  a0 b0



are tangent.

Theorem 13 (Problem 6.1.2 in MNZ). The number of Farey fractions a b of order n satisfying the inequalities 0 ≤ a

b ≤ 1 is 1 +

n

X

j=1

φ(j) and their sum is exactly half this value.

Proof. We want to prove the first part of the theorem. Let n be a posi- tive integer. Every Farey fraction a

b of order n must satisfy the inequalities

(27)

0 ≤ a

b ≤ 1 by Corollary 7, so the Farey fractions a

b of order n satisfying the inequalities 0 ≤ a

b ≤ 1 are precisely the Farey fractions of order n.

By Corollary 7 the Farey fractions of order n are precisely the reduced frac- tions a

b such that 0 ≤ a

b ≤ 1 and 0 < b ≤ n. For integers j such that 1 ≤ j ≤ n, define

A(j) := the number of reduced fractions a

j such that 0 ≤ a j ≤ 1.

Also let

F (n) := the number of Farey fractions of order n.

Then clearly F (n) =

n

X

j=1

A(j). It is clear that A(j) is the number of Farey fractions of order n with denominator j. We have that A(1) = 2, because

0

1 and 1

1 are the only reduced fractions with denominator 1 that lie between 0 and 1. The definition of A(j) can be rewritten by multiplying the last inequalities by j and using the definition of reduced fraction to obtain

A(j) = the number of fractions a

j such that gcd(a, j) = 1 and 0 ≤ a ≤ j.

Different values for a give different fractions a

j, so we can rewrite A(j) as A(j) = the number of integers a such that gcd(a, j) = 1 and 0 ≤ a ≤ j.

For j ≥ 2 we have that gcd(0, j) = j 6= 1, so for j ≥ 2 we can rewrite A(j) as

A(j) = the number of integers a such that gcd(a, j) = 1 and 1 ≤ a ≤ j

= φ(j).

So

F (n) =

n

X

j=1

A(j) = A(1) +

n

X

j=2

A(j) = 2 +

n

X

j=2

φ(j)

= 1 + φ(1) +

n

X

j=2

φ(j)

= 1 +

n

X

j=1

φ(j).

(28)

We want to prove the second part of the theorem. Let S(n) be the sum of the Farey fractions of order n. We want to show that S(n) = 1

2 1 +

n

X

j=1

φ(j)

! .

S(n) = X

a

b|ab is a Farey fraction of order n

a b.

If a

b is a Farey fraction of order n, then 1 −a

b = b − a

b is also a Farey fraction of order n. This is because if a

b is Farey fraction of order n then by Corollary 7 we have that 0 ≤ a ≤ b, so 0 ≤ b − a ≤ b and if gcd(a, b) = 1, then gcd(b − a, b) = 1 and hence by Corollary 7, b − a

b is also a Farey fraction of order n. If b ≥ 3, then it can’t be the case that a

b = b − a b if a

b is a Farey fraction of order n, since that would imply 2a = b, i.e. a|b, but also a = b

2 > 1, so that a is a common divisor of a and b that is strictly greater than 1, which means that gcd(a, b) 6= 1. So

S(n) = X

a

b|ab is a Farey fraction of order n and b≤2

a

b + X

a

b|ab is a Farey fraction of order n and b>2

a b.

The terms in the right sum can be paired up as  a

b, b − a b



. The terms in each pair add up to 1. If we assume n ≥ 2 then the only Farey fractions of order n with denominator ≤ 2 are 0

1, 1

1 and 1

2, so there are 1 +

n

X

j=1

φ(j) − 3

terms being added in the right sum, so there are 1 2

n

X

j=1

φ(j) − 2

!

pairs being

summed in the second sum. Thus the right sum equals 1 2

n

X

j=1

φ(j) − 2

! . The left sum equals 3

2, so S(n) = 3

2 +1 2

n

X

j=1

φ(j) − 2

!

= 1 2 1 +

n

X

j=1

φ(j)

! .

(29)

If n = 1, then S(n) = 0 1 + 1

1 = 1 = 1 2 1 +

1

X

j=1

φ(j)

!

, so the theorem holds for all n.

3 Rational Approximations

Theorem 14 (Theorem 6.7 in MNZ). Let a

b and c

d be Farey fractions of order n such that no other Farey fraction of order n lies between them. Then

a

b − a + c b + d

= 1

b(b + d) ≤ 1 b(n + 1)

and

c

d − a + c b + d

= 1

d(b + d) ≤ 1 d(n + 1). Proof. For the first formula we have

a

b −a + c b + d

=

a(b + d) − b(a + c) b(b + d)

= |ad − bc|

b(b + d)

= 1

b(b + d) by Theorem 1

≤ 1

b(n + 1). {since b + d ≥ n + 1 by Theorem 3}

The second formula is obtained in a similar way.

Theorem 15 (Theorem 6.8 in MNZ). Let n ∈ N>0 and x ∈ R. Then there is a rational number a

b such that 0 < b ≤ n and

x − a

b

≤ 1

b(n + 1).

(30)

Proof. Let n ∈ N>0 and x ∈ R. Let k be the unique integer such that 0 ≤ x + k < 1. There are 2 cases.

Case 1 : x + k has the same value as some Farey fraction of order n. Let p q be the Farey fraction of order n that x + k simplifies to. Let the rational number

a

b be p − k · q

q . Since p

q is a Farey fraction of order n, q ≤ n by Theorem 4.

So a

b is a rational number such that 0 < b ≤ n and

x − a

b =

x −p − k · q q

=

x + k − p − k · q

q − k · q q

= p q −p

q

= 0

≤ 1

b(n + 1). Case 2 : x + k does not have the same value as some Farey fraction of order n. It can’t be the case that x + k = 0, since 0

1 is a Farey fraction of order n. So 0 < x + k < 1. Let p

q be the greatest Farey fraction of order n that is smaller than x + k and let r

s be the smallest Farey fraction of order n that is greater than x + k. There is no Farey fraction of order n that lies between p q and r

s. Since p q and r

s are Farey fractions of order n, it follows from Theorem 4 that q ≤ n and s ≤ n. There are 2 cases now.

Case 2.1 : p + r

q + s ≤ x + k. Let the rational number a

b be r − k · s

s . We have

(31)

that a

b is a rational number such that 0 < b ≤ n and

x − a

b =

x − r − k · s s

=

x + k − r s

= r

s − (x + k)

≤ r

s − p + r q + s



since p + r

q + s ≤ x + k < r s



≤ 1

s(n + 1) by Theorem 14

= 1

b(n + 1).

Case 2.2 : p + r

q + s > x + k. Let the rational number a

b be p − k · q

q . We have that a

b is a rational number such that 0 < b ≤ n and

x −a

b =

x − p − k · q q

=

x + k − p q

p + r q + s − p

q



since p

q < x + k < p + r q + s



= p

q − p + r q + s

≤ 1

q(n + 1) by Theorem 14

= 1

b(n + 1).

Theorem 16 (Theorem 6.9 in MNZ). If ξ ∈ R \ Q, then there are infinitely many distinct rational numbers a

b such that

ξ − a

b < 1

b2.

(32)

Proof. Let ξ ∈ R \ Q. For each n ∈ N>0, let An := a

b ∈ Q

0 < b ≤ n ∧ ξ − a

b

≤ 1

b(n + 1)

 .

Let X := {An| n ∈ N>0} By Theorem 15, each An is non-empty, so that X is a set of non-empty sets. By the axiom of choice, there exists a choice function f : X → ∪X such that ∀A ∈ X : f (A) ∈ A. So let f : X → ∪X be a function such that ∀A ∈ X : f (A) ∈ A. For each n ∈ N>0, let an

bn

be the rational number f (An). Then

ξ − an bn

≤ 1

bn(n + 1)

< 1

b2n {since n + 1 > bn} for all n ∈ N>0.

We want to show there are infinitely many distinct an

bn. Assume for contra- diction there are only finitely many distinct an

bn. Then there are only finitely many distinct values for

ξ − an bn

. Let d := min

n∈N>0

ξ − an bn

. Clearly d ≥ 0 and d 6= 0, since d = 0 would imply that ξ = ak

bk for some k ∈ N>0, but that contradicts ξ being irrational. So d > 0. Then for all n ∈ N>0 we have that

d ≤

ξ − an bn

≤ 1

bn(n + 1)

≤ 1

n + 1. {since bn ≥ 1}

But if we let n be sufficiently large (n ≥ 1

d will work), then 1

n + 1 < 1 n ≤ d.

But then d < d, a contradiction.

Lemma 17 (Lemma 6.10 in MNZ). If x and y are positive integers then not both of the inequalities 1

xy ≥ 1

√5

 1 x2 + 1

y2



and 1

x(x + y) ≥ 1

√5

 1

x2 + 1 (x + y)2

 can hold.

(33)

Proof. We will rewrite the inequalities. We see that 1

xy ≥ 1

√5

 1 x2 + 1

y2



⇐⇒ 1

xy ≥ 1

√5 ·y2+ x2 (xy)2

⇐⇒ √

5xy ≥ y2+ x2 n

multiply both sides by √

5(xy)2o and

1

x(x + y) ≥ 1

√5

 1

x2 + 1 (x + y)2



⇐⇒ 1

x(x + y) ≥ 1

√5· (x + y)2+ x2 (x(x + y))2

⇐⇒ √

5x(x + y) ≥ (x + y)2+ x2. n

multiply both sides by √

5(x(x + y))2 o Assume for contradiction that both

5xy ≥ y2+ x2 (11)

and

5x(x + y) ≥ (x + y)2+ x2 (12) are true. By adding the inequalities (11) and (12) we get

5(x2+ 2xy) ≥ 3x2+ 2xy + 2y2

⇐⇒ 3x2+ 2xy + 2y2 ≤√

5(x2+ 2xy)

⇐⇒ (3 −√

5)x2− 2(√

5 − 1)xy + 2y2 ≤ 0

⇐⇒ (5 − 2√

5 + 1)x2− 4(√

5 − 1)xy + 4y2 ≤ 0 {multiply by 2}

⇐⇒ (2y − (√

5 − 1)x)2 ≤ 0

⇐⇒ 2y − (√

5 − 1)x = 0 {since squares are non-negative}

⇐⇒ √

5 = 2y + x

x , n

solve for √ 5o but that contradicts √

5 being irrational.

References

Related documents

Genom denna studie har min medvetenhet ökat kring matematiskt särbegåvade elever samt vilka hinder och möjligheter som finns för att möta dessa inom nuvarande

Beräkningarna i denna analys pekar på att de ACN-tal som finns angivna i litteraturen är för låga vid en undergrund med hög styvhet om hänsyn skall tas till påkänningen

Recent natura- listic research on commercial truck drivers indicates that the occurrence of crashes, near-crashes and safety critical events is correlated with handling the

(1992) Amount of fat at five measuring points; exact location of injection 50 patients with type 1 diabetes (250 measuring points) Ultrasound Descriptive 2 Engstr ¨om &amp;

Niss (2002) förklarade i sin artikel hur en del elever manipulerar ekvationer mekaniskt. Ele- verna uppfattar ekvationen som en signal till att utföra en del bestämda operationer,

Vi har valt att fokusera på hur konstruktörer, entreprenörer och tillverkare kan arbeta för att underlätta implementeringen av nya innovationer i stort men framförallt på hur

(Oster, 1995) Vårt sätt att beskriva detta hänsynstagande är att se det som att organisationerna förutom sina externa mål även har interna mål, vilka syftar till att skapa